库仑势的Fourier transform(update 16/Nov/2020)

问题

三维库仑势的形式为:

按动量Fourier展开:

求 $V(\vec{k})$ 。

求解

但 $e^{-ikr}$ 与 $e^{-ikr}$ 在 $+\infty$ 处发散。

使其收敛

为了解决发散问题,可在一开始令

所以

函数满足一致收敛,积分与极限才可交换顺序。

类似的变换

用同样的方法, 可以得到

用 Mathematica 可以验证.

Reference